Deduction of ##\forall x \in S \cup T (x \le b)## using first order logic rules.

  • Thread starter Thread starter Mr Davis 97
  • Start date Start date
Click For Summary
SUMMARY

The discussion centers on proving that the statement ##\forall x \in S \cup T (x \le b)## implies ##\forall s \in S (s \le b) \wedge \forall t \in T (t \le b)## using first-order logic. Participants confirm the logical equivalence of the two statements and demonstrate the deduction process through manipulations of first-order logic rules. The proof involves showing that if there exists an element in either set S or T that exceeds b, it contradicts the original statement, thus confirming the implication.

PREREQUISITES
  • Understanding of first-order logic principles
  • Familiarity with logical implications and equivalences
  • Knowledge of set theory, specifically unions and subsets
  • Ability to manipulate logical expressions and quantifiers
NEXT STEPS
  • Study the rules of inference in first-order logic
  • Explore proofs involving quantifiers and logical equivalences
  • Learn about set operations and their properties in mathematical logic
  • Investigate counterexamples in logic to understand implications better
USEFUL FOR

Mathematics students, logic enthusiasts, and educators looking to deepen their understanding of first-order logic and its applications in set theory and mathematical proofs.

Mr Davis 97
Messages
1,461
Reaction score
44

Homework Statement


Show that ##\forall x \in S \cup T (x \le b)## implies that ##\forall s \in S (s \le b) \wedge \forall t \in T (t \le b)##

Homework Equations

The Attempt at a Solution


How can I perform this deduction using the rules of first order logic? This is how far I can get:

##\forall x \in S \cup T (x \le b)##
##(x \in S \cup T \implies x \le b)##
##(x \in S \lor x \in T \implies x \le b)##
 
Physics news on Phys.org
I would write it ##s\in S \stackrel{def.}{\Longrightarrow} s \in S \cup T \stackrel{\text{ given cond. }}{\Longrightarrow} s \leq b## and ##t \in T \ldots##
 
  • Like
Likes Mr Davis 97
fresh_42 said:
I would write it ##s\in S \stackrel{def.}{\Longrightarrow} s \in S \cup T \stackrel{\text{ given cond. }}{\Longrightarrow} s \leq b## and ##t \in T \ldots##

I see, that makes sense. Just wondering, are the two statements I gave logically equivalent? That is, is there a way to derive one from the other using manipulations from first order logic?
 
Yes they are equivalent. We already have from left to right. From right to left is equivalent to non left implies non right. Assume ##\lnot [(\forall x)(x\in S\cup T)\Rightarrow(x \leq b)]##. That is ##(\exists x)(x \in S\cup T)\wedge (x>b)##. So ##x\in S## or ##x\in T## and still ##x>b## which is non right:
##[((\exists x)(x\in S) \wedge (x>b)) \vee ((\exists x)(x\in T) \wedge (x>b))] = \lnot [((\forall x)(x\in S)\Rightarrow(x\leq b)) \wedge ((\forall x)(x\in T)\Rightarrow(x\leq b))] ##
 
Question: A clock's minute hand has length 4 and its hour hand has length 3. What is the distance between the tips at the moment when it is increasing most rapidly?(Putnam Exam Question) Answer: Making assumption that both the hands moves at constant angular velocities, the answer is ## \sqrt{7} .## But don't you think this assumption is somewhat doubtful and wrong?

Similar threads

  • · Replies 1 ·
Replies
1
Views
1K
Replies
4
Views
1K
  • · Replies 7 ·
Replies
7
Views
3K
  • · Replies 3 ·
Replies
3
Views
1K
  • · Replies 43 ·
2
Replies
43
Views
4K
Replies
2
Views
1K
  • · Replies 12 ·
Replies
12
Views
3K
  • · Replies 3 ·
Replies
3
Views
1K
  • · Replies 1 ·
Replies
1
Views
2K
Replies
1
Views
2K